K
Khách

Hãy nhập câu hỏi của bạn vào đây, nếu là tài khoản VIP, bạn sẽ được ưu tiên trả lời.

4 tháng 5 2019

Ta có:

\(\frac{a}{1+b^2}=a-\frac{ab^2}{1+b^2}\)

Áp dụng BĐT AM-GM cho các số không âm, ta có:

\(1+b^2\ge2b\Rightarrow\frac{1}{1+b^2}\le\frac{1}{2b}\Rightarrow-\frac{1}{1+b^2}\ge-\frac{1}{2b}\)\(\Rightarrow-\frac{ab^2}{1+b^2}\ge-\frac{ab}{2}\)

\(\Rightarrow\frac{a}{1+b^2}\ge a-\frac{ab}{2}\)

CMTT: \(\frac{b}{1+c^2}\ge b-\frac{bc}{2};\frac{c}{1+a^2}\ge c-\frac{ca}{2}\)

\(\Rightarrow BĐT\ge a+b+c-\frac{ab+bc+ca}{2}\)\(=3-\frac{ab+bc+ca}{2}\)

Mặt khác ta có:

\(\left(a+b+c\right)^2\ge3\left(ab+bc+ca\right)\)

\(\Leftrightarrow3\ge ab+bc+ca\)\(\Leftrightarrow-\frac{3}{2}\le-\frac{ab+bc+ca}{2}\)

\(\Rightarrow BĐT\ge3-\frac{3}{2}=\frac{3}{2}\)(đpcm)

\(''=''\Leftrightarrow a=b=c=1\)

NV
19 tháng 6 2019

a/ BĐT sai, cho \(a=b=c=2\) là thấy

b/ \(VT=\frac{a^4}{a^2+2ab}+\frac{b^4}{b^2+2bc}+\frac{c^4}{c^2+2ac}\ge\frac{\left(a^2+b^2+c^2\right)^2}{\left(a+b+c\right)^2}=\frac{\left(a^2+b^2+c^2\right)\left(a^2+b^2+c^2\right)}{\left(a+b+c\right)^2}\)

\(VT\ge\frac{\left(a^2+b^2+c^2\right)\left(a+b+c\right)^2}{3\left(a+b+c\right)^2}=\frac{1}{3}\left(a^2+b^2+c^2\right)\)

Dấu "=" xảy ra khi \(a=b=c\)

c/ Tiếp tục sai nữa, vế phải là \(\frac{3}{2}\) chứ ko phải \(2\), và hy vọng rằng a;b;c dương

\(VT=\frac{a^2}{abc.b+a}+\frac{b^2}{abc.c+b}+\frac{c^2}{abc.a+c}\ge\frac{\left(a+b+c\right)^2}{abc\left(a+b+c\right)+a+b+c}\)

\(VT\ge\frac{9}{3abc+3}\ge\frac{9}{\frac{3\left(a+b+c\right)^3}{27}+3}=\frac{9}{\frac{3.3^3}{27}+3}=\frac{9}{6}=\frac{3}{2}\)

Dấu "=" xảy ra khi \(a=b=c=1\)

NV
23 tháng 6 2019

Ta có:

\(a^3+b^3+b^3\ge3ab^2\) ; \(b^3+c^3+c^3\ge3bc^2\) ; \(c^3+a^3+a^3\ge3ca^2\)

Cộng vế với vế \(\Rightarrow a^3+b^3+c^3\ge ab^2+bc^2+ca^2\)

\(\frac{a^5}{b^2}+\frac{b^5}{c^2}+\frac{c^5}{a^2}=\frac{a^6}{ab^2}+\frac{b^6}{bc^2}+\frac{c^6}{ca^2}\ge\frac{\left(a^3+b^3+c^3\right)^2}{ab^2+bc^2+ca^2}\ge\frac{\left(a^3+b^3+c^3\right)^2}{a^3+b^3+c^3}=a^3+b^3+c^3\)

31 tháng 7 2019

1. BĐT ban đầu

<=> \(\left(\frac{1}{3}-\frac{b}{a+3b}\right)+\left(\frac{1}{3}-\frac{c}{b+3c}\right)+\left(\frac{1}{3}-\frac{a}{c+3a}\right)\ge\frac{1}{4}\)

<=>\(\frac{a}{a+3b}+\frac{b}{b+3c}+\frac{c}{c+3a}\ge\frac{3}{4}\)

<=> \(\frac{a^2}{a^2+3ab}+\frac{b^2}{b^2+3bc}+\frac{c^2}{c^2+3ac}\ge\frac{3}{4}\)

Áp dụng BĐT buniacoxki dang phân thức 

=> BĐT cần CM

<=> \(\frac{\left(a+b+c\right)^2}{a^2+b^2+c^2+3\left(ab+bc+ac\right)}\ge\frac{3}{4}\)

<=> \(a^2+b^2+c^2\ge ab+bc+ac\)luôn đúng 

=> BĐT được CM

31 tháng 7 2019

2) \(a+b+c\le ab+bc+ca\le\frac{\left(a+b+c\right)^2}{3}\)\(\Leftrightarrow\)\(\left(a+b+c\right)^2-3\left(a+b+c\right)\ge0\)

\(\Leftrightarrow\)\(\left(a+b+c\right)\left(a+b+c-3\right)\ge0\)\(\Leftrightarrow\)\(a+b+c\ge3\)

ko mất tính tổng quát giả sử \(a\ge b\ge c\)

Có: \(3\le a+b+c\le ab+bc+ca\le3a^2\)\(\Leftrightarrow\)\(3a^2\ge3\)\(\Leftrightarrow\)\(a\ge1\)

=> \(\frac{1}{1+a+b}+\frac{1}{1+b+c}+\frac{1}{1+c+a}\le\frac{3}{1+2a}\le1\)

Dấu "=" xảy ra \(\Leftrightarrow\)\(a=b=c=1\)

2 tháng 7 2016

Bài 1:

Đặt \(a^2=x;b^2=y;c^2=z\)

Ta có:\(\sqrt{\frac{x}{x+y}}+\sqrt{\frac{y}{y+z}}+\sqrt{\frac{z}{z+x}}\le\frac{3}{\sqrt{2}}\)

Áp dụng BĐT cô si ta có:

\(\sqrt{\frac{x}{x+y}}=\frac{1}{\sqrt{2}}\sqrt{\frac{4x\left(x+y+z\right)}{3\left(x+y\right)\left(x+z\right)}\frac{3\left(x+z\right)}{2\left(x+y+z\right)}}\)

\(\le\frac{1}{2\sqrt{2}}\left[\frac{4x\left(x+y+z\right)}{3\left(x+y\right)\left(x+z\right)}+\frac{3\left(x+z\right)}{2\left(x+y+z\right)}\right]\)

Tương tự với \(\sqrt{\frac{y}{y+z}}\)và \(\sqrt{\frac{z}{z+x}}\)

Cộng lại ta được:

\(\frac{\sqrt{2}}{3}\left[\frac{x\left(x+y+z\right)}{\left(x+y\right)\left(x+z\right)}+\frac{y\left(x+y+z\right)}{\left(y+z\right)\left(y+x\right)}+\frac{z\left(x+y+z\right)}{\left(z+x\right)\left(z+y\right)}\right]+\frac{3}{2\sqrt{2}}\le\frac{3}{2\sqrt{2}}\)

Sau đó bình phương hai vế rồi

\(\Rightarrow\left(x+y\right)\left(y+z\right)\left(z+x\right)\ge8xyz\)đẳng thức đúng

Vậy...

Bài 2:

Trước hết ta chứng minh bất đẳng thức sau:

\(\frac{a}{4a+4b+c}+\frac{b}{4b+4c+a}+\frac{c}{4c+4a+b}\le\frac{1}{3}\)

Nhân cả hai vế bđt với 4(a+b+c)4(a+b+c) rồi thu gọn ta được bđt sau: 

\(\frac{4a\left(a+b+c\right)}{4a+4b+c}+\frac{4b\left(a+b+c\right)}{4b+4c+a}+\frac{4c\left(a+b+c\right)}{4c+4a+b}\)\(\le\frac{4}{3}\left(a+b+c\right)\)

\(\left[\frac{4a\left(a+b+c\right)}{4a+4b+}-a\right]+\left[\frac{4b\left(a+b+c\right)}{4b+4c+a}-b\right]+\left[\frac{4c\left(a+b+c\right)}{4c+4a+b}-c\right]\le\frac{a+b+c}{3}\)

\(\frac{ca}{4a+4b+c}+\frac{ab}{4b+4c+a}+\frac{bc}{4c+4a+b}\le\frac{a+b+c}{9}\)

Áp dụng bđt cauchy-Schwarz ta có \(\frac{ca}{4a+4b+c}=\frac{ca}{\left(2b+c\right)+2\left(2a+b\right)}\)\(\le\frac{ca}{9}\left(\frac{1}{2b+c}+\frac{2}{2a+b}\right)\)

Từ đó ta có:

\(\text{∑}\frac{ca}{4a+4b+c}\le\frac{1}{9}\text{∑}\left(\frac{ca}{2b+c}+\frac{2ca}{2a+b}\right)\)\(=\frac{1}{9}\left(\text{ ∑}\frac{ca}{2b+c}+\text{ ∑}\frac{2ca}{2a+b}\right)\)\(=\frac{1}{9}\left(\text{ ∑}\frac{ca}{2b+c}+\text{ ∑}\frac{2ab}{2b+c}\right)=\frac{a+b+c}{9}\)

Đặt VT=A rồi áp dụng bđt cauchy-Schwarz cho VT ta có 

\(T^2\le3\left(\frac{a}{4a+4b+c}+\frac{b}{4b+4c+a}+\frac{c}{4c+4a+b}\right)\)\(\le3\cdot\frac{1}{3}=1\Leftrightarrow T\le1\)

Dấu = xảy ra khi a=b=c 

c bạn tự làm nhé mình mệt rồi :D

2 tháng 7 2016

- Ôi má ơi, má patient dử dậy :)

10 tháng 7 2019

Bài 1: \(a+\frac{1}{b\left(a-b\right)}=\left(a-b\right)+b+\frac{1}{b\left(a-b\right)}\)

Áp dụng BĐT Cauchy cho 3 số dương ta thu được đpcm (mình làm ở đâu đó rồi mà:)

Dấu "=" xảy ra khi a =2; b =1 (tự giải ra)

Bài 2: Thêm đk a,b,c >0.

Theo BĐT Cauchy \(\frac{a^2}{b^2}+\frac{b^2}{c^2}\ge2\sqrt{\frac{a^2}{c^2}}=\frac{2a}{c}\). Tương tự với hai cặp còn lại và cộng theo vế ròi 6chia cho 2 hai có đpcm.

Bài 3: Nó sao sao ấy ta?

18 tháng 11 2019

1. Vai trò a, b, c như nhau. Không mất tính tổng quát. Giả sử \(a\ge b\ge0\)

\(ab+bc+ca=3\). Do đó \(ab\ge1\)

Ta cần chứng minh rằng \(\frac{1}{1+a^2}+\frac{1}{1+b^2}\ge\frac{2}{1+ab}\left(1\right)\)

\(\frac{2}{1+ab}+\frac{1}{1+c^2}\ge\frac{3}{2}\left(2\right)\)

Thật vậy: \(\left(1\right)\Leftrightarrow\frac{1}{1+a^2}-\frac{1}{1+ab}+\frac{1}{1+b^2}-\frac{1}{1+ab}\ge0\\ \Leftrightarrow\left(ab-a^2\right)\left(1+b^2\right)+\left(ab-b^2\right)\left(1+a^2\right)\ge0\\ \Leftrightarrow\left(a-b\right)\left[-a\left(1+b^2\right)+b\left(1+a^2\right)\right]\ge0\\ \Leftrightarrow\left(a-b\right)^2\left(ab-1\right)\ge0\left(BĐT:đúng\right)\)

\(\left(2\right)\Leftrightarrow c^2+3-ab\ge3abc^2\\ \Leftrightarrow c^2+ca+bc\ge3abc^2\Leftrightarrow a+b+c\ge3abc\)

BĐT đúng, vì \(\left(a+b+c\right)^2>3\left(ab+bc+ca\right)=q\)

\(ab+bc+ca\ge3\sqrt[3]{\left(abc\right)^2}\)

Nên \(a+b+c\ge3\ge3abc\)

Từ (1) và (2) ta có \(\frac{1}{1+a^2}+\frac{1}{1+b^2}+\frac{1}{1+c^2}\ge\frac{3}{2}\)

Dấu ''='' xảy ra \(\Leftrightarrow a=b=c=1\)

18 tháng 11 2019

Áp dụng BĐT Cauchy dạng \(\frac{9}{x+y+z}\le\frac{1}{x}+\frac{1}{y}+\frac{1}{z}\), ta được

\(\frac{9}{a+3b+2c}=\frac{1}{a+c+b+c+2b}\le\frac{1}{9}\left(\frac{1}{a+c}+\frac{1}{b+c}+\frac{1}{2b}\right)\)

Do đó ta được

\(\frac{ab}{a+3b+2c}\le\frac{ab}{9}\left(\frac{1}{a+c}+\frac{1}{b+c}+\frac{1}{2b}\right)=\frac{1}{9}\left(\frac{ab}{a+c}+\frac{ab}{b+c}+\frac{a}{2}\right)\)

Hoàn toàn tương tự ta được

\(\frac{bc}{2a+b+3c}\le\frac{1}{9}\left(\frac{bc}{a+b}+\frac{bc}{b+c}+\frac{b}{2}\right);\frac{ac}{3a+2b+c}\le\frac{1}{9}\left(\frac{ac}{a+b}+\frac{ac}{b+c}+\frac{c}{2}\right)\)

Cộng theo vế các BĐT trên ta được

\(\frac{ab}{a+3b+2c}+\frac{bc}{b+3c+2a}+\frac{ca}{c+3a+2b}\le\frac{1}{9}\left(\frac{ac+bc}{a+b}+\frac{ab+ac}{b+c}+\frac{bc+ab}{a+c}+\frac{a+b+c}{2}\right)=\frac{a+b+c}{6}\)Vậy BĐT đc CM

ĐẲng thức xảy ra khi và chỉ khi a = b = c >0

18 tháng 9 2015

a)Bạn đặt A = a/ (1 + a^2). => A + a^2A = a => a^2A - a + A = 0. ta có delta = 1 - 4A^2 ( gọi ẩn số là a). => để pt có nghiệm <=> 1 - 4A^2 >= 0 => để phương trình có nghiệm => 1 - 4A^2 >= 0 => 1 >= 4A^2 => A =< 1/2. => max A = 1/2. bạn giải tương tự B = b/(1+b^2), C = c/(1 + c^2) rồi cộng vào nhau là ra ngay thôi. Đây là cách giải bằng delta. 

b)bạn có (a^2 - b^2)/c = ((a+b)(a-b))/c >= (c + c)(a-b)/c = 2(a - b). Bạn có c =< b ( theo đề bài) = > c + b =< 2b => (c + b) =<2b => (c + b)/b <= 2 => (c + b)/a <= 2. từ đó ta có (c^2 - b^2)/a = (c -b )(c + b)/a >= 2(c - b).

chứng minh tương tự:(a + c)/b > 1 => (a^2 - c^2)/b >= a - c.( sr ngại gõ lắm) => cộng 3 vế ta được đpcm

17 tháng 8 2019

\(VT=\frac{1}{a^2+1}+\frac{1}{b^2+1}+\frac{1}{c^2+1}\)

\(=1-\frac{a^2}{a^2+1}+1-\frac{b^2}{b^2+1}+1-\frac{c^2}{c^2+1}\)

\(=3-\left(\frac{a^2}{a^2+1}+\frac{b^2}{b^2+1}+\frac{c^2}{c^2+1}\right)\)

Áp dụng bất đẳng thức Cauchy :

\(VT\ge3-\left(\frac{a^2}{2a}+\frac{b^2}{2b}+\frac{c^2}{2c}\right)=3-\left(\frac{a}{2}+\frac{b}{2}+\frac{c}{2}\right)\)

\(=3-\frac{a+b+c}{2}=3-\frac{3}{2}=\frac{3}{2}\)

Dấu "=" xảy ra \(\Leftrightarrow a=b=c=1\)

17 tháng 8 2019

\(ab+ac+bc\le a^2+b^2+c^2\\ \Rightarrow3\left(ab+ac+bc\right)\le a^2+b^2+c^2+2\left(ab+ac+bc\right)\\ \Rightarrow3\left(ab+ac+bc\right)\le\left(a+b+c\right)^2=9\\ \Rightarrow ab+ac+bc\le3\\ \Rightarrow2\left(ab+ac+bc\right)\le6\)

Áp dụng BDT Cô-si với 3 số dương:

\(\Rightarrow\frac{1}{a^2+1}+\frac{1}{b^2+1}+\frac{1}{c^2+1}\ge\frac{9}{a^2+1+b^2+1+c^2+1}\\ =\frac{9}{a^2+b^2+c^2+3}=\frac{9}{a^2+b^2+c^2+6-3}\\ \ge\frac{9}{a^2+b^2+c^2+2\left(ab+ac+bc\right)-3}=\frac{9}{\left(a+b+c\right)^2-3}=\frac{9}{6}=\frac{3}{2}\)

Dấu "=" xảy ra khi a=b=c=1

5 tháng 11 2016

Câu 1: a)

b) Áp dụng Bđt Holder ta có:

\(\Rightarrow9\left(a^3+b^3+c^3\right)\ge\left(a+b+c\right)^3\)

\(\Rightarrow\frac{a^3+b^3+c^3}{3}\ge\frac{\left(a+b+c\right)^3}{27}=\left(\frac{a+b+c}{3}\right)^3\)(đpcm)

Dấu = khi a=b=c

Câu 2:

Áp dụng Bđt \(\frac{1}{x}+\frac{1}{y}\ge\frac{4}{x+y}\)ta có:

\(\frac{1}{a+1}+\frac{1}{b+1}\ge\frac{4}{a+b+1+1}=\frac{4}{3}\)(Đpcm)

Dấu = khi \(a=b=\frac{1}{2}\)

Câu 3:

Áp dụng Bđt \(\frac{1}{x}+\frac{1}{y}+\frac{1}{z}\ge\frac{9}{x+y+z}\)ta có:

\(\frac{1}{a}+\frac{1}{b}+\frac{1}{c}\ge\frac{9}{a+b+c}=9\left(a+b+c=1\right)\)(Đpcm)

Dấu = khi \(a=b=c=\frac{1}{3}\)

Câu 4: nghĩ sau